LSAT and Law School Admissions Forum

Get expert LSAT preparation and law school admissions advice from PowerScore Test Preparation.

 Administrator
PowerScore Staff
  • PowerScore Staff
  • Posts: 8916
  • Joined: Feb 02, 2011
|
#40929
Complete Question Explanation
(The complete setup for this game can be found here: lsat/viewtopic.php?t=1864)

The correct answer choice is (E)

If M is used in the first year, then from the third rule T is used in the fourth year. L cannot be used in the second or third years, and must appear in the fifth or sixth years:
PT59_Game_#4_#21_diagram 1.png
From the second rule, the VW block must be used in the second and third years, although the exact order cannot be determined. Regardless, N, which is the remainder of the WN split-block, must then be used in the fifth or sixth year, rotating with L. This information results in the following diagram:
PT59_Game_#4_#21_diagram 2.png
Accordingly, L cannot be used in the third year, and answer choice (E) cannot be true and is correct.
You do not have the required permissions to view the files attached to this post.

Get the most out of your LSAT Prep Plus subscription.

Analyze and track your performance with our Testing and Analytics Package.